Which equations should Amber graph?

Which Equations Should Amber Graph?

Answers

Answer 1

Answer:

amber should graph the second equation


Related Questions

For which value of b can the expression x2 + bx + 18 be factored?

Answers

Answers:

b = -19b = -11b = -9b = 19b = 11b = 9

====================================================

Explanation:

Here are all the ways to multiply to 18 when using integers only:

-1*(-18) = 18-2*(-9) = 18-3*(-6) = 181*18 = 182*9 = 183*6 = 18

Sum each pair of factors to find out a possible value of b.

-1 + (-18) = -19-2 + (-9) = -11-3 + (-6) = -91 + 18 = 192 + 9 = 113 + 6 = 9

Therefore, the possible values of b are

b = -19b = -11b = -9b = 19b = 11b = 9

which are the final answers.

----------------------

An example:

Let's say b = 11. This would mean [tex]x^2+bx+18[/tex] becomes [tex]x^2+11x+18[/tex]

It would factor to [tex](x+2)(x+9)[/tex] since it was stated earlier that:

2+9 = 11

2 * 9 = 18

You can use the FOIL rule, distributive property, or the box method to confirm that [tex]x^2+11x+18 = (x+2)(x+9)[/tex] is a true equation for all real numbers x.

This same idea applies for the other values of b.

----------------------

If you're curious why this works, consider multiplying the two factors (x+p) and (x+q)

Use the FOIL rule to get [tex](x+p)(x+q) = x^2+qx+px+pq = x^2+(p+q)x + pq[/tex]

The middle term [tex](p+q)x[/tex] has the components add to the coefficient, while those same two components multiply to get the last term. This is why when factoring we're looking for two numbers that multiply to 18, and also add to the value of b (which in the case of the last example was 11).

Your friend writes an equation of the line shown. Is your friend correct?
Student work is shown. A line is graphed on a coordinate plane. The line passes through the points at ordered pair (0,-2) and ordered pair (4,0). Your friend writes an equation of the line shown. Is your friend correct? Their equation was y = 1/2x + 4.

Answers

Answer:

No. The correct equation is

y= 1/2 x - 2

Step-by-step explanation:

See picture

(-6) x 8
What is this ?????

Answers

Answer:

-(6) x 8

ans: - 48

I believe that is the ans:

Answer:

-48

Step-by-step explanation:

Integer multiplication:

   When we multiply a negative integer by a positive integer , we will get a negative integer.

(-6) * 8 = (-48)

negative integer  * negative integer = positive integer

(-2) * (-3) = 6

negative integer * positive integer  = negative integer

(-2) * 3 = (-6)

Positive integer *negative integer =  negative integer

2 * (-3) = (-6)

Identify a horizontal or vertical stretch or compression of the function f(x)=\sqrt(x) by observing the equation of the function g(x)=\sqrt((3)/(2)x)

kind of urgent lol

Answers

By applying the concept of transformation, the transformed function g(x) = √[(3/2) · x] is the consequence of applying a stretch factor of 3/2 on the parent function f(x) = √x.

How to compare two functions by concepts of transformation

In this question we have a parent function g(x) = √[(3/2) · x] and a transformed function f(x) = √x. Transformations are operations in which parent functions are modified in their relationships between inputs and outputs.

In this case, the difference between f(x) and g(x) occurred because of the application of a operation known as vertical stretch, defined below:

f(x) = g(k · x), k > 0     (1)

Where k is the stretch factor. There is a compression for 0 ≤ k < 1.

By applying the concept of transformation, the transformed function g(x) = √[(3/2) · x] is the consequence of applying a stretch factor of 2/3 on the parent function f(x) = √x. (Right choice: C)

To learn more on transformations: https://brainly.com/question/11709244

#SPJ1

The equation of a circuits in the form: (in the picture)

If the circle is centered in Quadrant I, what must be true of h and k?
(Answer choices in the picture as well)

Answers

Answer: h>0 and k>0

Step-by-step explanation:

If the circle is centered in Quadrant I, then both the x and y coordinates of the center are positive.

This means that h>0 and k>0.

The graph shows the height of an elevator over a period of time.

On which intervals could the elevator be traveling from a higher floor to a lower floor?
A. Between A and B and between C and D
B. Between B and C and between D and E
C. Between E and F and between G and H
D. Between F and G

Answers

C. Between E and F and between G and H the elevator could be travelling from a higher floor to a lower floor.

We have to justify all the options with respect to the graph :-

Option A : Between A and B, the elevator would be travelling from a lower floor to a higher floor. Also between C and D, the elevator would be travelling from a lower floor to a slightly higher floor.

So, this option is not correct.

Option B : Between B and C, the elevator wouldn't go higher. And also the same would happen in between D and E.

So, this option is not correct.

Option C : Between E and F, the elevator would be travelling from higher floor to lower and also between G and H.

So, this option is correct.

Option 4 : Between F and G, there would not have much changes. So it will be omitted.

So, this option is not correct.

Learn more about graph here :

https://brainly.com/question/4025726

#SPJ10

PLS HELP! Which statement is true about the given function?

Answers

The statement which is true about the given graph is  that f(x)<0 in the interval (-∞,3) which is option 3.

Given Graph of a function

We have to choose a statement which is correct about the function whose graph is given.

The graph of a function tells us about the domain and range of the function. The values on y axis are the codomain of the function and the values on x axis are the values of domain.

When we see the graph we can find that x=-∞ to x=3 the values are negative means when we put the values of x less than 0 we will get negative number. For example if we put the value of x=-1, we will get -20 which is a negative number.

Hence the third statement is true for the given graph which is that it has values less than zero in the interval (-∞,3).

Learn more about function at https://brainly.com/question/10439235

#SPJ10

Easy 40 ptS!!!!!! Functions transformations

Answers

Answer:

Step-by-step explanation:

The population of a small industrial town was 12 910 in 2000. Each year, the population
decreases by an average of 5%. Estimate the population in the year 2020. Round to the nearestwhole number.

Answers

The population in the year 2020 is 4628

How to determine the population?

The given parameters are:

Initial, a = 12910

Rate, r = 5%

Since the population decreases, then we make use of an exponential decay function.

This is represented as:

f(n) = a * (1 - r)^n

So, we have:

f(n) = 12910 * (1 - 5%)^n

Evaluate the difference

f(n) = 12910 * 0.95^n

2020 is 20 years from 2000.

So, we have:

f(20) = 12910 * 0.95^20

Evaluate

f(20) = 4628

Hence, the population in the year 2020 is 4628

Read more about exponential functions at:

https://brainly.com/question/14355665

#SPJ1

Math again yay!...Ew math

Answers

Answer:

The graph of g(x) is wider.

Step-by-step explanation:

Parent function:

[tex]f(x)=x^2[/tex]

New function:

[tex]g(x)=\left(\dfrac{1}{2}x\right)^2=\dfrac{1}{4}x^2[/tex]

Transformations:

For a > 0

[tex]f(x)+a \implies f(x) \: \textsf{translated}\:a\:\textsf{units up}[/tex]

[tex]f(x)-a \implies f(x) \: \textsf{translated}\:a\:\textsf{units down}[/tex]

[tex]\begin{aligned} y =a\:f(x) \implies & f(x) \: \textsf{stretched/compressed vertically by a factor of}\:a\\ & \textsf{If }a > 1 \textsf{ it is stretched by a factor of}\: a\\ & \textsf{If }0 < a < 1 \textsf{ it is compressed by a factor of}\: a\\\end{aligned}[/tex]

[tex]\begin{aligned} y=f(ax) \implies & f(x) \: \textsf{stretched/compressed horizontally by a factor of} \: a\\& \textsf{If }a > 1 \textsf{ it is compressed by a factor of}\: a\\ & \textsf{If }0 < a < 1 \textsf{ it is stretched by a factor of}\: a\\\end{aligned}[/tex]

If the parent function is shifted ¹/₄ unit up:

[tex]\implies g(x)=x^2+\dfrac{1}{4}[/tex]

If the parent function is shifted ¹/₄ unit down:

[tex]\implies g(x)=x^2-\dfrac{1}{4}[/tex]

If the parent function is compressed vertically by a factor of ¹/₄:

[tex]\implies g(x)=\dfrac{1}{4}x^2[/tex]

If the parent function is stretched horizontally by a factor of ¹/₂:

[tex]\implies g(x)=\left(\dfrac{1}{2}x\right)^2[/tex]

Therefore, a vertical compression and a horizontal stretch mean that the graph of g(x) is wider.

4. Try It #4 Write the point-slope form of an equation of a line that passes through the points (-1,3) and (0,0). Then rewrite it in the slope-intercept form.

Answers

Answer:

Point-slope form of equation of a line that passes from (-1,3) and (0,0) is given as y-3=-3(x+1).

Slope-intercept form of equation is given as y=-3x.

Step-by-step explanation:

In the question, it is given that the line passes from (-1,3) and (0,0).

It is asked to write the point-slope form of the equation and rewrite it as slope-intercept form.

Step 1 of 2

Passing point of line is (-1,3).

Hence, [tex]$x_{1}=-1$[/tex] and

[tex]$$y_{1}=3 \text {. }$$[/tex]

Also, Passing point of line is (0,0).

Hence, [tex]$x_{2}=0$[/tex] and

[tex]$$y_{2}=0 \text {. }$$[/tex]

Substitute the above values to find the slope of line which is given by [tex]$m=\frac{y_{2}-y_{1}}{x_{2}-x_{1}}$[/tex]

[tex]$$\begin{aligned}m &=\frac{y_{2}-y_{1}}{x_{2}-x_{1}} \\m &=\frac{0-3}{0-(-1)} \\m &=\frac{-3}{1} \\m &=-3\end{aligned}$$[/tex]

Hence, slope of the line is -3

Step 2 of 3

It is obtained that m=-3

[tex]$y_{1}=3$[/tex]

and [tex]$x_{1}=-1$[/tex]

Substitute the above values in point-slope form of equation given by [tex]$y-y_{1}=m\left(x-x_{1}\right)$[/tex]

[tex]$y-y_{1}=m\left(x-x_{1}\right)$\\ $y-3=-3(x-(-1)$\\ $y-3=-3(x+1)$[/tex]

Hence, point-slope form of equation given as y-3=-3(x+1).

Step 3 of 3

Solve y-3=-3(x+1) to write it as slope-intercept form given by y=mx+c

[tex]$y-3=-3(x+1)$\\ $y-3=-3 x-3$\\ $y=-3 x-3+3$\\ $y=-3 x$[/tex]

Hence, slope-intercept form of equation is given as y=-3x.

An equation is shown. the
Which describes n?
n = 1 ÷17

Answers

1
-
17

decimal form 0.05882352
SOLVING

[tex]\Large\maltese\underline{\textsf{A. What is Asked}}[/tex]

An equation is shown. What is the value of n? [tex]\bf{n=1:17}[/tex] is shown

[tex]\Large\maltese\underline{\textsf{B. This problem has been solved!}}[/tex]

[tex]\bf{n=1:17}[/tex] | divide

[tex]\bf{n=\dfrac{1}{17}[/tex]

[tex]\rule{300}{1.7}[/tex]

[tex]\bf{Result:}[/tex]

          [tex]\bf{=n=\dfrac{1}{17}}[/tex]

[tex]\boxed{\bf{aesthetic \not101}}[/tex]

While the goal of hypothesis testing is to test a claim, the goal of estimation is to estimate a

Answers

The estimation goal is to estimate a population parameter. The estimation process uses sample statistics.

What is the goal of hypothesis testing?

The goal of hypothesis testing is testing to claim whether it is right or wrong.

The hypothesis testing also uses statistics to determine whether or not a treatment has an effect.

What is the goal of estimation?

The estimation goal is to estimate a population parameter. The estimation is used to determine how much effect a treatment has.

To estimate a parameter, a sample statistics of the parameter is used.

Thus, the estimation is to estimate a population parameter.

Learn more about estimation and hypothesis testing here:

https://brainly.com/question/20367153

#SPJ4

HEEEEEEEEEELP i dont get it

Answers

The ordered pair is (-9, -25) and the word statement is if x is equal to -9, then the value of h(x) is -25

Functions and tables

From the given table, f(x) = y means that the corresponding value of y given a value x.

For the function h(-9), we need to find the equivalent value of h(x) when x is -9. Hence h(-9) is -25

The ordered pair is (-9, -25) and the word statement is if x is equal to -9, then the value of h(x) is -25

Learn more on functions and values here: https://brainly.com/question/2284360

#SPJ1

pls help lol !!! i am unsure about this

Answers

The component form of the vectors shown is (-6, -5)

Difference of vectors

In order to determine the component of the vectors shown, we will subtract the coordinate points from both each other.

Given the vector coordinates on the line. as (-5, -3) and (1, 2). Take the difference;

Difference = [(-5-1), (-3-2])

Difference = (-6, -5)

Hence the  component form of the vectors shown is (-6, -5)

Learn more on vectors here: https://brainly.com/question/15519257

#SPJ1

can you teach me this​

Answers

The percentiles are: P40 = 33, P70 = 45, and P89 = 54.1, while the quartiles are Q1 = 28.5 and Q3 = 47.5

How to determine the percentiles?

The sorted dataset is:

22, 22, 24, 25, 26, 27, 27, 30, 30, 32,

33, 33, 35, 37, 38, 38, 40, 42, 44, 44,

45, 47, 48, 48, 49, 52, 55, 58, 62, 68

The number of data elements is:

N = 30

The 40th percentile (P40)

This is calculated using:

Element = (40% * N)th

So, we have:

Element = (40% * 30)th

Evaluate

Element = 12th

The 12th element is 33

Hence, the value of P40 is 33

The 70th percentile (P70)

This is calculated using:

Element = (70% * N)th

So, we have:

Element = (70% * 30)th

Evaluate

Element = 21st

The 21st element is 45

Hence, the value of P70 is 45

The 89th percentile (P89)

This is calculated using:

Element = (89% * N)th

So, we have:

Element = (89% * 30)th

Evaluate

Element = 26.7th

The element is calculated as:

Element = 26th + 0.7 * (27th - 26th)

So, we have:

Element = 52 + 0.7 * (55 - 52)

Element = 54.1

Hence, the value of P89 is 54.1

How to determine the quartiles?

The 1st quartile (Q1)

This is calculated using:

Element = (1/4 * N)th

So, we have:

Element = (1/4 * 30)th

Evaluate

Element = 7.5th

The element is calculated as:

Element = 7th + 0.5 * (8th - 7th)

So, we have:

Element = 27 + 0.5 * (30- 27)

Element = 28.5

Hence, the value of Q1 is 28.5

The 3rd quartile (Q3)

This is calculated using:

Element = (3/4 * N)th

So, we have:

Element = (3/4 * 30)th

Evaluate

Element = 22.5th

The element is calculated as:

Element = 22nd + 0.5 * (23rd - 22nd)

So, we have:

Element = 47 + 0.5 * (48- 47)

Element = 47.5

Hence, the value of Q3 is 47.5

The 5th quartile (Q5)

There is no such thing as Q5 i.e. the 5th quartile

Read more about quartiles and percentiles at:

https://brainly.com/question/20340210

#SPJ1

Puja limbu did 8 out of 10 math problems and raju lama did 11 out of 15 similar maths problems.express the number of problems solved by each of them in fractions and identify who did better performance.

Answers

Puja Limbu had the better performance than Raju Limbu.

Such question are generally solved using percentages.

In mathematics, a percentage is a number or ratio that represents a fraction of 100. It is often denoted by the symbol "%" or simply as "percent" or "pct." For example, 35% is equivalent to the decimal 0.35, or the fraction.

Percentage of Raju limba = [tex]\frac{11}{15}[/tex] X 100 = 73.3%

Percentage of Puja Limba = [tex]\frac{8}{10}[/tex] x 100 = 80%

As percentage of Puja Limba is greater than that of Raju Limba .

Hence performance of Puja Limba is better

Learn more about percentages here :

https://brainly.com/question/27986974

#SPJ4

AC=
Help me please!! Thanks so much

Answers

Answer:

AC = 6√3 in

Step-by-step explanation:

Finding the length of the chord:

Join OC. Now ΔAOC is an isosceles triangle as OA = OC =radius.

 ∠A = ∠C = 30.

 ∠A + ∠C +  ∠AOC = 180 {angle sum property of traingle}

 30 + 30 + ∠AOC  = 180°

                 ∠AOC = 180 -60

                 ∠AOC = Ф = 120°

Find the length of radius using the bellow formula.

         [tex]\sf \boxed{\bf Arc \ length = \dfrac{\theta}{180}\pi r}[/tex]

            Ф = 120°

                Arc length = 4π

            [tex]\sf 4\pi =\dfrac{120}{180}*\pi *r\\\\ r =\dfrac{4\pi * 180}{120*\pi }\\\\ r = 6 \ in[/tex]

[tex]\sf \boxed{\bf chord \ length = 2rSin \ \dfrac{\theta}{2}}[/tex]

                         [tex]\sf b = 2*6*Sin \ \dfrac{120}{2}\\\\ b = 2 *6 * Sin \ 60^\circ\\\\ b = 2 * 6 * \dfrac{\sqrt{3}}{2}\\\\ \b = 6\sqrt{3}[/tex]

[tex]\sf \boxed{\bf AC = 6\sqrt{3} \ in}[/tex]

Which of the following sets represents the range of the diagram below? 2 3. 00 4 4 5 5 f(x) O A. {2, 4, 5) OB. (1, 3, 4, 5} OC. (2, 3, 4, 5, 6, 7} D. {1, 2, 3, 4, 5)​

Answers

Answer:

D

Step-by-step explanation:

the answer is D because the range is the lowest possible value up to the highest possible value and when listed it doesn't repeat

Find the measure of angle ABD.

Answers

Answer:

x=58

Step-by-step explanation:

Exterior angle of one interior angle in a triangle is equal to the sum of the other two remote interior angles. (Ext. Angle Th.) So, 3x-32=84+x

2x=116

x=58

Paco pago $105.0 por 10 tacos cual es el precio de cada taco si todos tienen el mismo precio

Answers

The price of each taco is $10.5

What is unitary method?

We can solve this question by unitary method.
The unitary method is a method of finding the value of one unit and then finding the value of the required number of units. While solving a problem it is important to recognize the units and values.


In this question, 10 tacos cost $105.
Let's represent the cost of 1 taco as [tex]x[/tex]
10 tacos =$ 105
1 taco = [tex]x[/tex]
10[tex]x[/tex] = 105
[tex]x[/tex] = [tex]\frac{105}{10}[/tex]

[tex]x[/tex] = 10.5
Each taco cost $10,5
To read more about unitary method,
https://brainly.com/question/1955459
#SPJ4

Two hikers are wandering through heavy woods with walkie talkies. The walkie talkies have a range of 100 yards. From their starting point, they head off at an angle of 109°10' of each other. Hiker 1 walks 0.24 miles per hour, hiker 2 walks 0.17 miles per hour. If each continues to go straight, how long will it be before they can no longer communicate?

Answers

Answer: after t hours, the distance d between the hikers is

d^2 = (.24t)^2 + (.17t)^2 - 2(.24t)(.17t)cos109.167°

so, find t when d = 0.0568182

10 minutes

Step-by-step explanation:

prove the equation (2x+5)2 = 4x (x + 5) +25

Answers

Answer:  x = -5/2 and x = -3/2

Step-by-step explanation:

(2x + 5)2 = 4x (x + 5) +25

   4x + 10 = 4x² + 20x + 25

   [minus 4x on both sides.]

            10 = 4x² + 16x + 25

   [minus 10 on both sides.]

            0 = 4x² + 16x + 15

ac = 4(15) = 60,then find the factors that add up to 16, which is 6 and 10.

            0 = 4x² + 6x + 10x + 15

            0 = 2x(2x + 3) + 5(2x + 3)

            0 = (2x + 5)(2x + 3)

2x + 5 = 0                                 2x + 3 = 0

       2x = -5                                    2x = -3

         x = -5/2                                    x = -3/2

[tex]\huge\text{Hey there!}[/tex]

[tex]\textbf{Assuming you meant: }\mathsf{(2x + 5)^2 = 4x(x + 5) + 25}[/tex]

[tex]\textbf{If so, simplify both sides of your equation you're working with}[/tex]

[tex]\mathsf{ 4x^2 + 20x + 25 = 4x^2 + 20x + 25}[/tex]

[tex]\textbf{SUBTRACT }\rm{\bf 4x^2}\text{ \bf to BOTH of the SIDES}[/tex]

[tex]\mathsf{4x^2 + 20x + 25 - 4x^2 = 4x^2 + 20x + 25 - 4x^2}[/tex]

[tex]\textbf{Simplify it!}[/tex]

[tex]\mathsf{20x + 25 = 20x + 25}[/tex]

[tex]\textbf{SUBTRACT 20x to BOTH of the SIDES}[/tex]

[tex]\mathsf{20x + 25 - 20x = 20x + 25 - 20x}[/tex]

[tex]\large\textbf{SIMPLIFY IT! (as well)}[/tex]

[tex]\mathsf{25 = 25}[/tex]

[tex]\textbf{SUBTRACT 25 to BOTH of the SIDES}[/tex]

[tex]\mathsf{25 - 25 = 25 - 25}[/tex]

[tex]\textbf{Lastly, SIMPLIFY THAT!}[/tex]

[tex]\textbf{We get: }\mathsf{0 = 0}[/tex]

[tex]\large\textsf{This means that your \boxed{\textsf{solutions}} are \bf REAL NUMBERS.}[/tex]

[tex]\huge\textsf{Therefore, your answer should be: }\\\boxed{\mathsf{All\ \underline{\underline{REAL\ NUMBERS}}\ are\ solutions.}}}\huge\checkmark[/tex]

[tex]\huge\text{Good luck on your assignment \& enjoy your day!}[/tex]

~[tex]\frak{Amphitrite1040:)}[/tex]

the total drive is 450 miles, mila drives x mph for the first 200 miles, for the remaining distance she drives x + 30 mph

Answers

First part of the trip:
Time taken= 200/x

Second part of the trip:
Time taken= 250/(×+30)

Average:
Time taken= 450/30
Time taken= 15 hours

200/x + 250/(×+3) = 15
-15x^2+405x+600=0
x=28.40805152

3 quick algebra 1 questions for 50 points!

Only answer if you know the answer, tysm for the help!

Answers

Step-by-step explanation for each question:

For Question 6, the range of a function is all the possible outputs of the function. Since the function can only take the inputs 0, 4, and 7, we can just plug in each into the formula and find their corresponding outputs.

g(0) = 0² - 9 = 0 - 9 = -9

g(4) = 4² - 9 = 16 - 9 = 7

g(7) = 7² - 9 = 49 - 9 = 40

Therefore the only possible outputs of function g, or the range, is {-9, 7, 40}.

For question 4, the input t is a given time, and h(t) is the height of the football at that time.

Hence, h(2.5) is the height of the football (in feet) at 2.5 seconds. The value 2.5 can be plugged into the function [tex]-16t^2+58t+2[/tex] to get the height. This gives us

[tex]-16(2.5)^2 + 58(2.5) + 2[/tex]

[tex]-16(6.25) + 58(2.5) + 2[/tex] [Squaring 2.5]

[tex]-100 + 145 + 2[/tex] [Multiplying]

[tex]47[/tex] [Combining all terms]

We find that the height of the football at 2.5 seconds is 47 feet.

For Question 5, the table of values show all the possible values x can be (or the domain), and what the output of the function f(x) would give for each.

A) f(-3) = 5, as the row with -3 for x has -5 for y.

B) f(0) = 0, as the row with 0 for x has 0 for y.

C) f(1) = -3, as the row with 1 for x has -3 for y.

The range of the function will be -9,7 and 40.

What is the difference between domain and range?

The domain denotes all potential x values, while the range denotes all possible y values.

Given equation;

g(x) = x²-9

The range of the given domain is found by putting the values one by one in the above equation as;

g(x) = x²-9

a)For x = 0

g(x) = 0²-9

g(x) =-9

b)For x =4

g(x) = 4²-9

g(x) =16-9

g(x) = 7

c)For x =7

g(x) = 7²-9

g(x) =49-9

g(x) = 40

Hence, the range of the function will be -9,7 and 40.

To learn more about domain and range, refer to https://brainly.com/question/1632425

#SPJ1

The students in Ms. Yuri's class reported the number of hours they watched television last week.



Which is the interquartile range of the number of hours last week that the students watched television?

Answers

The interquartile range is 12.

What is the interquartile range?

The interquartile range is the difference between the third quartile and the first quartile. The first quartile is the first line on the box while the third quartile is the third line on the box.

First quartile = 11

Third quartile = 23

Interquartile range = 23 - 11  = 12

To learn more about interquartile range, please check:  https://brainly.com/question/3966385

#SPJ1

Joaquin deposits $1,000 into an account that accrues 4% annual interest continuously compounded. What is the value of his investment after 3 years to the nearest cent?

Answers

Deposit: $1,000

Annual interest: 4% = 0.04

Years: 3

For this type of question, when the question asks you to "continuously compound", you use this formula: [tex]Pe^{rt}[/tex]

Solving:

[tex]1000e^{(0.04)(3)} \\1000e^{0.12} \\=1127.50[/tex]

The value of Joaquin's investment after 3 years = 1,127.50$

Select the correct answer.
Convert sqrt3 + i to polar form.

Answers

The polar form of any complex number can be written as

[tex]z = |z| e^{i\arg(z)}[/tex]

where [tex]\arg(z)[/tex] is the argument of [tex]z[/tex], i.e. the angle it makes with the positive real axis in the complex plane.

If [tex]z=\sqrt3+i[/tex], then [tex]z[/tex] has modulus

[tex]|z| = \sqrt{\left(\sqrt3\right)^2 + 1^2} = \sqrt4 = 2[/tex]

and argument

[tex]\arg(z) = \tan^{-1}\left(\dfrac1{\sqrt3}\right) = \dfrac\pi6[/tex]

Then

[tex]\sqrt3 + i = 2e^{i\frac\pi6} = 2 \left(\cos\left(\dfrac\pi6\right) + i \sin\left(\dfrac\pi6\right)\right)[/tex]

what does y =4 in coordinates look like

Answers

When y = 4 x = 0. It should be on the y axis (vertical) 4 units up from the middle (0,0)

Suppose a large shipment of laser printers contained 12% defectives. If a sample of size 406 is selected, what is the probability that the sample proportion will be greater than 13%

Answers

The probability that the sample proportion will differ from the population proportion by less than 6% is 0.992.

According to the Central limit theorem, if from an unknown population large samples of sizes n > 30, are selected and the sample proportion for each sample is computed then the sampling distribution of sample proportion follows a Normal distribution.

The mean of this sampling distribution of sample proportion is:

цр = р

The standard deviation of this sampling distribution of sample proportion is:

бр = √ρ(1-ρ)÷n

The information provided is:

ρ = 0.22

ⁿ = 276

As the sample size is large, i.e. n = 276 > 30, the Central limit theorem can be used to approximate the sampling distribution of sample proportion.

Compute the value of P(р-p<0.06) as follows:

P(р-p<0.06) = P(р-p ÷ бp<0.06 ÷√0.22(1 - 0.22) ÷ 276

= P ( Z < 2.41 )

= 0.99202

≈ 0.992

Thus, the probability that the sample proportion will differ from the population proportion by less than 6% is 0.992.

Learn more about the probability at

https://brainly.com/question/24756209

#SPJ4

Other Questions
please please help meee!!!What trigonometric function should be used when the opposite and adjacent side ofa triangle are known?a) CSCb) tanc) tan^-1d) sin^-1e) sinf) cos^-1g) COSh) sec Discuss your thoughts on the overall lab design. Did it help you understand the concepts better, or did it raise more questions? Do you think you could have designed a better experiment? If so, explain how and then discuss it with your classmates. Share some of your knowledge with them or learn a little more about the experiment you just did. Read paragraphs 1 and 2 from the passage.(1) Developed by NASA's Jet Propulsion Laboratory in Pasadena, California, the Deep Space Atomic Clock is a serious upgrade to the satellite-based atomic clocks that, for example, enable the GPS on your phone.(2) Ultimately, this new technology could make spacecraft navigation to distant locations like Mars more autonomous. But what is an atomic clock? How are they used in space navigation, and what makes the Deep Space Atomic Clock different? Read on to get all the answers.How does the author unfold the idea that the Deep Space Atomic Clock is a serious upgrade to satellite-based atomic clocks?A. The author compares the new technology to the old technology, delineating exactly what makes the new technology superior.B. The author presents a series of facts and makes an appeal to authority to convince readers of their accuracy.C. The author follows up her claim by asking a series of questions that readers will have to continue reading to get answered.D. The author follows up her claim by describing how the Deep Space Atomic Clock could make space travel better. Twenty years ago, you won a state lottery, and you received $15,000 at the end of each of the next 10 years, and $20,000 at the end of years 11 through 15. Since you did well in FIN 3403, you decided to invest each of your lottery payments. If you earned 9% per year, how much do you have at the end of year 20 fully simplify 7h + 2h + 5 - h + h How many moles of ammonia are in 0.40 l of a 6.00 m ammonia solution? if this solution were diluted to 1.00 l, what would be the molarity of the resulting solution? n The Number Devil, the number devil's character helps to build on the point of the story bychallenging Robert's negative view of numbers.getting himself into a difficult situation.asking Robert the origin of his pretzel tale.finding a unique way to solve a problem. What is the length of segment AC?1111.4135.7 If Elin delivers a check payable to her order to First Bank without signing it and receives cash, the transfer is an assignment, not a negotiation. If Elin delivers a check payable to her order to First Bank without signing it and receives cash, the transfer is an assignment, not a negotiation. True False Which function is shown in the graph?O f(x)=1090.2*O f(x) = logxO f(x) = logxO f(x) = log0x Find the y-intercept of the line by substituting 0 in for x and solving for y. y = 4x -12 According to Verbrugge and Jette (1994), the main pathway of disability emphasizes the relations between pathology and ______________. subtract 6x 2 7x2 from 6x^{2}-2x+6 Read the excerpt from The Importance of Being Earnest.Jack. It pains me very much to have to speak frankly to you, Lady Bracknell, about your nephew, but the fact is that I do not approve at all of his moral character. I suspect him of being untruthful. [Algernon and Cecily look at him in indignant amazement.]Lady Bracknell. Untruthful! My nephew Algernon? Impossible! He is an Oxonian.How does Lady Bracknell's defense of Algernon advance the plot?It leads to Cecily withdrawing her acceptance of Algernon's proposal.It leads to Cecily becoming upset with Algernon for lying. It causes Jack to refuse to allow Cecily to become engaged to Algernon. It causes Jack to reveal that Algernon broke into Jack's home by pretending to be his brother. The Allowance for Doubtful Accounts T-account will have the ______ on the credit side. Multiple choice question. sales discounts and allowances write-offs of specific customers sales on account estimated bad debts from the adjusting entry QUESTION 4Malik and Zahara are neighbors involved in a boundary dispute. Malik sues Zahara, and Zahara receives written notice that Malik hasfiled a lawsuit against her. Zahara has receivedOa an answer.Oba default judgment.4 hat drawing technique is each the artworks below an example of?How is the medium made that is used for each technique?Picture 1: A black and white painting of bamboo branches and leaves with Chinese characters written in the bottom right hand corner. Picture 2: Two men wearing black suits and black top hats. They are facing the same direction and one of the men is leaning over the other's shoulder to speak in his ear. They are standing in a room with other people. Susan needs material for her school project. She buys 3.75 yards of material at $5.72 a yard. What is the total cost of the material? Round to the nearest cent. The Theater Club draws a tree on the set background. The plan for the size of the tree is shown below. What is the approximate area they willhave to paint to fill in this tree?5 ft3 ft0.2 ftThe top of the tree is a triangle. Its area is approximately ft.The second layer of the tree is a trapezoid. Its area is approximatelyThe trunk of the tree is a rectangle. Its area is approximatelyThe total area of the tree is approximatelyft.All rights reserved.Resetft.ft.Next 5. Greg used a sensor to measure the speed of a moving car at differenttimes. At each time, the sensor measured the speed of the car in bothmiles per hour and kilometers per hour. The table below shows her results.Based on the results, which statement describes the relationship betweenthe m, speed of the car in miles per hour, and k, the speed of the car inkilometers per hour?The relationship is not proportional because the distance of m to k is constant.The relationship is proportional because the difference of m to k is constant.The relationship is proportional because the ratio of m to k is constant.The relationship is not proportional because the ratio of m to k is constant.This is the table